Difference between revisions of "1966 AHSME Problems/Problem 24"

(Solution)
(Solution)
Line 5: Line 5:
  
 
== Solution ==
 
== Solution ==
<math>\fbox{B}</math>
+
If we change the base of <math>Log_M{N}</math> to base <math>N</math>, we get <math>\frac{1}{log_N{M}}= Log_N{M}</math>. Multiplying both sides by <math>Log_N{M}</math>, we get <math>Log_N{M}^2=1</math>. Since <math>N\not = M</math>, <math>Log_N{M}=-1</math>. So <math>N^{-1}=M \rightarrow \frac{1}{N} = M \rightarrow MN=1</math>. So the answer is <math>\boxed{B}</math>.
  
 
== See also ==
 
== See also ==

Revision as of 16:48, 2 January 2020

Problem

If $Log_M{N}=Log_N{M},M \ne N,MN>0,M \ne 1, N \ne 1$, then $MN$ equals:

$\text{(A) } \frac{1}{2} \quad \text{(B) } 1 \quad \text{(C) } 2 \quad \text{(D) } 10 \\ \text{(E) a number greater than 2 and less than 10}$

Solution

If we change the base of $Log_M{N}$ to base $N$, we get $\frac{1}{log_N{M}}= Log_N{M}$. Multiplying both sides by $Log_N{M}$, we get $Log_N{M}^2=1$. Since $N\not = M$, $Log_N{M}=-1$. So $N^{-1}=M \rightarrow \frac{1}{N} = M \rightarrow MN=1$. So the answer is $\boxed{B}$.

See also

1966 AHSME (ProblemsAnswer KeyResources)
Preceded by
Problem 23
Followed by
Problem 25
1 2 3 4 5 6 7 8 9 10 11 12 13 14 15 16 17 18 19 20 21 22 23 24 25 26 27 28 29 30
All AHSME Problems and Solutions

The problems on this page are copyrighted by the Mathematical Association of America's American Mathematics Competitions. AMC logo.png